pls answer i will mark brainliest

Pls Answer I Will Mark Brainliest

Answers

Answer 1

Answer:

Part one:

In terms of price;

[tex]2x + \frac{5}{2} y = 19 \\ \\ 4x + 5y = 38 - - - (equation \: \: 1)[/tex]

In terms of people, each person will get one snack:

[tex]x + y = 8 - - - (equation \: \: 2)[/tex]

Part two:

[tex]in \: equation \: 2 : \\ x = 8 - y \\ substitute \: for \: x \: in \: 1 : \\ 4(8 - y) + 5y = 38 \\ 32 - 4y + 5y = 38 \\ y = 6 \\ \\ therefore : x = 2[/tex]

Sam bought 2 bags of chips


Related Questions

Compute the standard deviation of the following set of data to the nearest whole number:
8, 12, 15, 17, 18
14
03
04
13

Answers

Answer:

Step-by-step explanation:

The standard deviation is the square root of the variance, and the variance is found by using the mean. So we will do that first. I will use the population variance as opposed to the sample variance since our set of numbers is small.

The mean: 8 + 12 + 15 + 17 + 18 = 70 and divide that by 5 to get

[tex]\bar{x}=14[/tex] and use this to find the variance in the formula:

[tex]s^2=\frac{\sum(x_i-\bar{x})^2}{n}[/tex] it is a bit difficult to enter that formula in correctly, but here's how it works mathematically:

[tex]s^2=\frac{(8-14)^2+(12-14)^2+(15-14)^2+(17-14)^2+(18-14)^2}{5}[/tex]

Squaring this ensures us that we don't end up with zero, which would be useless.

[tex]s^2=\frac{36+4+1+9+16}{5}[/tex] so

[tex]s^2=13.2[/tex] which means that the standard deviation is

s = 3.633

(If you do it with n-1 = 4 in the denominator of the variance, you get a standard deviation of 4.062)

PLS HELP PICS OF QUESTION ATTACHED!

Answers

Sorry but you didn’t post the question

Which of the following function pairs are inverses?

Answers

Answer:

(A)

x= 5y-3

(x+3)/5

Step-by-step explanation:

Answer:

its A

Step-by-step explanation:

i took the test lol

Plesssse help me with this

Answers

i think it’s the fourth one

If x=(p-3) and y=(p+3),show that xy+9=psquare​

Answers

Step-by-step explanation:

Answer with explanation is in the attachment

Hope it is helpful to you

HELP ASAP ILL GIVE BRAINLIST



Construct the equation of the graph given. Label the intercept and categorize this graph a horizontal or vertical line.

Answers

Answer:

Horizontal line

Step-by-step explanation:

uehdjd

SOMEONE HELP PLEASE I NEED ANSWER

Answers

T=55+6w is the equation

at the end of 8 weeks you do
T=55+6x8
which equals 103$

Answer:

T = 55 + 6w and $103

Step-by-step explanation:

We have already saved $55, so that is our constant. We gain $6 per week, so that will be our w value

T = 55 + 6w

Solving for T when w = 8 weeks

T = 55 + 6(8)

T= 55 + 48

T = $103

PLEASE HELP !!!!!!!!!!!!!

Answers

Answer:

the 3rd won for sure

Step-by-step explanation:

hope I helped

Answer:

C

Step-by-step explanation:

The coordinates are all in (x, y) form.

X is how left or right

y is how tall.

If you just look for the individual coordinates given, you will see that all of the Coordinates in C are on the graph.

I hope this helps!

What are the domain and range of the inequality y < sqrt x+3+1

I don't get it I try to solve it but I just didn't get .

Answers

Given:

The inequality is:

[tex]y<\sqrt{x+3}+1[/tex]

To find:

The domain and range of the given inequality.

Solution:

We have,

[tex]y<\sqrt{x+3}+1[/tex]

The related equation is:

[tex]y=\sqrt{x+3}+1[/tex]

This equation is defined if:

[tex]x+3\geq 0[/tex]

[tex]x\geq -3[/tex]

In the given inequality, the sign of inequality is <, it means the points on the boundary line are not included in the solution set. Thus, -3 is not included in the domain.

So, the domain of the given inequality is x>-3.

We know that,

[tex]\sqrt{x+3}\geq 0[/tex]

[tex]\sqrt{x+3}+1\geq 0+1[/tex]

[tex]y\geq 1[/tex]

The points on the boundary line are not included in the solution set. Thus, 1 is not included in the range.

So, the domain of the given inequality is y>1.

Therefore, the correct option is A.

At an amusement park, 63 out of 70 tickets sold were discount tickets.
What percentage of the tickets were discount tickets?

Answers

Answer:

90%

Step-by-step explanation:

63/70 = 0.9 = 90%

Answer:

63 out of 70

63×10÷70=9

9%

Which set of numbers does NOT represent the lengths of the sides of a triangle? A.) (7,5,6)
B.) (9,12,19)
C.) (7,18,11)
D.) (6,8,11)

Answers

Answer: B and C

Step-by-step explanation:

the triangle inequality theorem states that any 2 sides of a triangle must be greater than the measure of the third side. now with this rule, let's apply it to all options!

option a:

7 + 5 > 6 .. yes!

6+ 7 > 5 .. yes!

5 + 6 > 7 .. yes!

option b:

9 + 12 > 19 .. yes!

12 + 19 > 9 .. no!

19 + 9 > 12 .. no!

option c:

7 + 11 > 18 .. no!

11 + 18 > 7 .. no!

18 + 7 > 11 .. yes!

option d:

6 + 8 > 11 .. yes!

8 + 11 > 6 .. yes!

11 + 6 > 8 .. yes!

i did this topic years ago so please let me know if there are any mistakes! hope this helped, and good luck! ◝(ᵔᵕᵔ)◜

In a bowl of fruit, there are green grapes and black grapes in the ratio 3 : 4

If there are 24 green grapes, how many black grapes are there?

Answers

Answer:

32

Step-by-step explanation:

Green : black

3             4

We have 24 green

3+8=24

so  multiply each side by 8

Green : black

3*8         4*8

24            32

There are 32 black grapes

Answer:

32

Step-by-step explanation:

By Question :-

Green grapes: Black grapes= 3:4

Let :-

Number of black grapes = x

Therefore,

GG/ BG = 3/4 24/x = 3/4 x = 24 × 4/3x = 8 × 4x = 32

Please help 33 points

Answers

Answer: it’s 111

Step-by-step explanation:

PLEASE HELP 30 POINTS

Find the value of x in the triangle below.
804
155
55°
o
75
80°
Ο Ο
85

Answers

Answer:

75°

Step-by-step explanation:

First, we need to know that the interior angle sum of a triangle is always 180°.

Now, to find the missing angle aside from x, we need to complete the sum 180-155=25, seeing 155° lies on a straight line. Remembering the interior angle sum of a triangle is always 180°, it is thus clear that 25+80+x=180. If we complete this equation, x=75.

Hope this helps!

Elisa runs a bubble bath for her daughter. The bathtub is 17 inches deep and fills at a rate of 3 inches of water every 5 minutes.
Just after turning on the water, Elisa’s doorbell rings. It’s her neighbor bringing over a package that was mistakenly left on her doorstep. They talk for 26 minutes, then Elisa returns to check on the bathtub. Did the tub overflow while she was talking to her neighbor, or did she return in time?

Answers

Answer:

Elisa return in time. :)

Step-by-step explanation:

It is given that the bathtub is 17 inches deep.

The water fills the tub at a rate of 3 inches of water for every 5 minutes.

Elisa spent 26 minutes with neighbor.

Let's find whether the tub overflow or did she return in time.

In 5 minutes  water filled is 3 inches

In 10 minutes water filled is 6 inches

In 15 minutes water filled is 9 inches

In 20 minutes water filled is 12 inches

In 25 minutes water filled is 15 inches

In 26 minutes water filled in is 15+1/3 inches , which is same as 15.33 inches.

But the deep of tub is 17 inches.

We could say, she return in time.

Please HELP! ):
The measure of an interior angle of a regular polygon is eight times the measure of one of its exterior angles. How many sides does the polygon have?

Answers

A polygon has 18 sides :)

One condition for performing a hypothesis test is that the observations are independent. Mark is going to take a sample from a population of 400 students.How many students will Marta have to sample without replacement to treat the observations as independent?

Answers

Answer:

40

Step-by-step explanation:

Given :

The number of the observations in the population is N = 400

There is a [tex]10\%[/tex] rule of assuming the independence.

Now, according to the rule, if we sample without replacement then, any sample of sizes less than or equal to [tex]10\%[/tex] of the population is considered independent.

[tex]10\%[/tex] of population = [tex]10\%[/tex] of 400

                              [tex]$=\frac{10}{100} \times 400$[/tex]

                             = 40

Any sample of sizes less than or equal to 40 will be independent for the given population if it is sampled without replacement.

So, the answer is 40.

So, Mark have a sample of 40 students without replacement to treat the observations as independent.

Answer: 40

Step-by-step explanation:

Can someone help me with this math homework please!

Answers

Answer:

1 ) - 0.04

2 ) 1

3 ) - 0.04

4 ) Less of 0.04 gallons of fuel for every 1 mile.

Divide ( 9/11) ÷ (3/2) ​

Answers

Answer:

Its your answer ......

hope it will help u

9/11 divided by 3/3= 9/11 x 2/3= 8/33

0=(x-8)^2-100 Solve the problem using any type of way.

Answers

0 = (x-8)^2 -100

Add 100 to both sides

100 = (x-8)^2

Take the square root of both sides

10 = (x-8)

Remove the parentheses

10 = x-8

Add 8 to both sides

18 = x

Answer: x = 18

99999+999=????????????????????????????

Answers

Answer:

100998 is the answer.

I hope this help u.

Answer:

100998

Step-by-step explanation:

₁ ₁ ₁ ₁ ₁

99999

+   999

100998

you can also think of it as

99999(+1) + 999(-1)

100000+998

100998

Drag the tiles to the correct boxes to complete the pairs.
Find the missing the coordinates.
(2, -2)
(-5, -5)
(2, 1)
(-1, -1)
A(-5, -1), B(1, -1), and C(1, -5) are
three vertices of rectangle ABCD. What
are the coordinates of vertex D?
arrowRight
A(2, 5), B(6, 5), and C(6, 1) are
three vertices of square ABCD. What are
the coordinates of vertex D?
arrowRight
A(2, 3), B(7, 3), and C(7, -2) are
three vertices of square ABCD. What are
the coordinates of vertex D?
arrowRight
A(-1, 4), B(7, 4), and C(7, -1) are
three vertices of rectangle ABCD. What
are the coordinates of vertex D?
arrowRight

Answers

Answer:

i think its 24

Step-by-step explanation:

2=2

88 POINTS PLS HELP!!!

Answers

Answer:

aw no me la seee lo.sientooo no puedooo

Janes but a small clectric car and recorded the distance it waveled. The table below shows the distance waveled (nt during the st 4 seco
Bapsed Time Distance Traveled
(seconds) feet)
1
62
2
125
3
186
243
Which of the following equations represents the relationship between the distance traveled and the clapsed time?
Of=52-5
52-
+=52
52

Answers

Answer:

50

Step-by-step explanation:

Which number is greater than 234,781,114?
A. 3.425 x 10 to the power of 6
B. 3.425 x 10 to the power of 8
C. 3.425 x 10 to the power of 7
D. 3.425 x 10 to the power of 5

Answers

Answer:

I think its B.

Step-by-step explanation:

sorry if i am wrong.

Can someone help me with this math homework please!

Answers

Answer:

I believe its (-3,-2)...........

In triangle ABC, AC=13, BC=84, and AB=85. Find the measure of angle C

Answers

Given:

In triangle [tex]ABC, AC=13, BC=84, AB=85[/tex].

To find:

The measure of angle C.

Solution:

Law of Cosines:

[tex]\cos C=\dfrac{a^2+b^2-c^2}{2ab}[/tex]

[tex]\cos C=\dfrac{BC^2+AC^2-AB^2}{2(BC)(AC)}[/tex]

Substituting the given values, we get

[tex]\cos C=\dfrac{(84)^2+(13)^2-(85)^2}{2(84)(13)}[/tex]

[tex]\cos C=\dfrac{7056+169-7225}{2184}[/tex]

[tex]\cos C=\dfrac{0}{2184}[/tex]

[tex]\cos C=0[/tex]

Taking cos inverse on both sides, we get

[tex]C=\cos^{-1}(0)[/tex]

[tex]C=90^\circ[/tex]

Therefore, the measure of angle C is 90 degrees.

Answer:

eason

Step-by-step explanation:

boy

why did u read this

why did u read this

why did u read this

why did u read this

why did u read this

why did u read this

why did u read this

why did u read this

why did u read thiswhy did u read this

why did u read thiswhy did u read thiswhy did u read thiswhy did u read thiswhy did u read thiswhy did u read thiswhy did u read thiswhy did u read thiswhy did u read thiswhy did u read thiswhy did u read thiswhy did u read thiswhy did u read thiswhy did u read thisAnswer:

eason

Step-by-step explanation:

boy

why did u read this

why did u read this

why did u read this

why did u read this

why did u read this

why did u read this

why did u read this

why did u read this

why did u read thiswhy did u read this

why did u read thiswhy did u read thiswhy did u read thiswhy did u read thiswhy did u read thiswhy did u read thiswhy did u read thiswhy did u read thiswhy did u read thiswhy did u read thiswhy did u read thiswhy did u read thiswhy did u read thiswhy did u read thisAnswer:

eason

Step-by-step explanation:

boy

why did u read this

why did u read this

why did u read this

why did u read this

why did u read this

why did u read this

why did u read this

why did u read this

why did u read thiswhy did u read this

why did u read thiswhy did u read thiswhy did u read thiswhy did u read thiswhy did u read thiswhy did u read thiswhy did u read thiswhy did u read thiswhy did u read thiswhy did u read thiswhy did u read thiswhy did u read thiswhy did u read thiswhy did u read thisAnswer:

eason

Step-by-step explanation:

boy

why did u read this

why did u read this

why did u read this

why did u read this

why did u read this

why did u read this

why did u read this

why did u read this

why did u read thiswhy did u read this

why did u read thiswhy did u read thiswhy did u read thiswhy did u read thiswhy did u read thiswhy did u read thiswhy did u read thiswhy did u read thiswhy did u read thiswhy did u read thiswhy did u read thiswhy did u read thiswhy did u read thiswhy did u read thisAnswer:

eason

Step-by-step explanation:

boy

why did u read this

why did u read this

why did u read this

why did u read this

why did u read this

why did u read this

why did u read this

why did u read this

why did u read thiswhy did u read this

why did u read thiswhy did u read thiswhy did u read thiswhy did u read thiswhy did u read thiswhy did u read thiswhy did u read thiswhy did u read thiswhy did u read thiswhy did u read thiswhy did u read thiswhy did u read thiswhy did u read thiswhy did u read thisAnswer:

eason

Step-by-step explanation:

boy

why did u read this

why did u read this

why did u read this

why did u read this

why did u read this

why did u read this

why did u read this

why did u read this

why did u read thiswhy did u read this

why did u read thiswhy did u read thiswhy did u read thiswhy did u read thiswhy did u read thiswhy did u read thiswhy did u read thiswhy did u read thiswhy did u read thiswhy did u read thiswhy did u read thiswhy did u read thiswhy did u read thiswhy did u read this

see screenshot below​

Answers

Answer:

B only

Step-by-step explanation:

well to solve this question we can consider factor theorem which states that if a Polynomial f(x) is divided by a monomial x-a then x-a is a factor of f(x) if and only if f(a)=0

and to figure "a " we can consider the following equation:

[tex] \displaystyle x - a = 0[/tex]

substitute the given value of a:

[tex] \displaystyle x - 2= 0[/tex]

therefore,

[tex] \displaystyle x = 2[/tex]

option-1:

plugin the value of x to the function A(x):

[tex] \displaystyle A(2) = {2}^{3} + {2}^{2} + 4[/tex]

simplify which yields:

[tex] \displaystyle A(2) = 16[/tex]

since A(2)≠0 x-2 is not a factor of A(x)

option-2:

similarly substitute the value of x to the function B(x) which yields:

[tex] \displaystyle B(2) = {2}^{3} - {2}^{} - 6[/tex]

simplify:

[tex] \displaystyle B(2) =0[/tex]

so, B(2)=0 Hence x-2 is a factor of x³-x-6

option-3:

substitute the value of x to the function C(x)

[tex] \displaystyle C(2) = {2}^{4} + 3.{2}^{} - 10[/tex]

simplify:

[tex] \displaystyle C(2) =12[/tex]

as C(2)≠0 x-2 is not a factor of C(x)

option-4:

likewise plugin the value of x to the function D(x):

[tex] \displaystyle D(2) = {2}^{4} - 2.{2}^{3} - 2[/tex]

simplify and that yields:

[tex] \displaystyle D(2) = - 2[/tex]

therefore,D(2)≠0 hence,x-2 isn't a factor of D(-2)

[tex]\boxed{\large{\bold{\textbf{\textsf{{\color{blue}{Answer}}}}}}:)}[/tex]

The factor

x-2=0x=2

we have find the value of x .

Now,

we have to select all the polynomial which has x-2 As a factor.

According to the question,

option A:-

[tex]A(x)=x^3+x^2+4\\\\A(2)=(2)^3+(2)^2+4\\\\=8+4+4\\\\16≠0[/tex]

so it is not a factor.

Option B:-

[tex]B(x)=x^3-x-6\\\\=B(2)=(2)^2-2-6\\\\0=0[/tex]

so it is a factor

Option C:-

[tex]C(x)=x^4+3x-10\\\\=C(2)=(2)^4+3(2)-10\\\\=16+6-10\\\\12≠0[/tex]

so it is not a factor

option D:-

[tex]D(x)=x^4-2x^3-2\\\\D(2)=(2)^4-2(2)^3-2\\\\=16-16-2\\\\-2≠0[/tex]

so it is not a factor

therefore:-

The option B is correct.

Find the value of x. Round the length to the nearest tenth.

Answers

Answer:

[tex]x=opposite\: side\: of \:22[/tex]

[tex]16=hypotenese[/tex]

[tex]sin22=opp/hyp[/tex]

[tex]sin22=\frac{x}{16}[/tex]

[tex]\Longrightarrow x=16\times sin22[/tex]

[tex]x=5.99[/tex]

[tex]x=6.0[/tex]

[tex]OAmalOHopeO[/tex]

Find the size of the angles marked by letters in the following diagram.​

Answers

Answer:

x = 52

y = 19

Step-by-step explanation:

The degree measure of a straight line is (180) degrees. One can use this to find the measure of the angle that is adjacent to the angle with a measure of (142) degrees. Form an equation and solve for the unknown,

142 + (unknown angle) = 180

unknown angle = 38

The inscribed angle theorem states that twice the measure of an angle with its vertex of the circumference (outer edge) of a circle is equal to the measure of the surrounding arc. Therefore, one can state the following:

(2)(unknown angle) = (surrounding arc)

(2)(38) = (surrounding arc)

76 = surrounding arc

The central angles theorem states that the measure of an angle whose vertex is the center of the circle is equal to the measure of the surrounding arc. Therefore, one can state the following:

m<O = (surrounding arc)

m<O = 76

The radius is the distance from the center of the circle to the circumference of the circle. All radii in a circle are congrunet. Therefore the triangle with angle (x) and vertex (O) is an isosceles triangle, as two of its sides are radii, and are thus congruent. One property of an isosceles triangle is the base angles theorem. This theorem states that the angles opposite the congruent sides in an isosceles triangle are congruent. Moreover, the sum of angles in any triangle is (180) degrees. Therefore, one can make the following statement:

m<O + x + x = 180

76 + 2x = 180

x = 52

Finally, one can use the property that the sum of angles in any triangle is (180) degrees to make the following statement:

(x + 19) + (x + y) + (38) = 180

52 + 19 + 52 + y + 38 = 180

161 + y = 180

y = 19

x=52°

y=19°

Answer:

Solution given:

<OCA=19°[base angle of isosceles triangle]

Since ∆AOC is similar to ∆ BOC

y=19°[corresponding angle of a similar triangle are equal]

<OAB=<OBA=x[base angle of isosceles triangle]

again

<A+<B=180°[exterior angle of a triangle is equal to the sum of two opposite interior angle]

x+19+x+19=142°

2x=142°-38°

x=104/2

x=52°

Other Questions
When the audience finally sees Paris and Juliet interact in Act IV of Romeo andJuliet, how does Juliet act around Paris?O A. FearfulB. IndifferentC. LovingD. Hateful What is desertification?a.method of using desert land for agricultureb.the process of turning arable land into desertsC.drying up of stream beds and lakesd.the expansion of current deserts land areas HELPP!!What is the approximate sum of this series? Help!!!! Give a simple overview of the process of photosynthesis. What news does Harriet bring when she visits Elwood? Why do the Americans gives great value to indivdualism and time Structure of DNA:Question 6What is the function of DNA? Select one:To encode instructions for traits.To help the cell move.To store energy for the cell.To protect the cell. I need help ASAP please the UK that follows the best democratic norms in the world has no written constitution. why? how do elasticity ang flexibility differ? A website where I can find informations about: The effectiveness of state programs to support Children with Autism Spectrum Disorder in the UK? cho v d v truyn thng dng nc i i vs gi nc ca t tin Math nation section 2 A(n) _____ is a hydrocarbon in which one or more hydrogen atoms is replaced by a -COOH and a -NH2 group. aldehyde carboxylic acid alcohol amino acid need help fixing my mistake:19. Haces la tarea para tu profesora?my answer: S, la hago para ella. teachers message: para tu profesora is the IO, which IOP do we use? Where does it go? Does it need to become se? can anyone help me please ? According to Newton's third law of physics, who would have strong muscles: a long-distance runner or a runner who jumps hurdles? Question Two please help Your firm has $45.0 million invested in accounts receivable, which is 90 days of net revenues. If this value could be reduced to 50 days, what annual increase in income would your firm realize if the increase in cash could be invested at 7.5 percent need help with this!!!